Last visit was: 23 Apr 2024, 20:13 It is currently 23 Apr 2024, 20:13

Close
GMAT Club Daily Prep
Thank you for using the timer - this advanced tool can estimate your performance and suggest more practice questions. We have subscribed you to Daily Prep Questions via email.

Customized
for You

we will pick new questions that match your level based on your Timer History

Track
Your Progress

every week, we’ll send you an estimated GMAT score based on your performance

Practice
Pays

we will pick new questions that match your level based on your Timer History
Not interested in getting valuable practice questions and articles delivered to your email? No problem, unsubscribe here.
Close
Request Expert Reply
Confirm Cancel
SORT BY:
Date
Tags:
Show Tags
Hide Tags
Math Expert
Joined: 02 Sep 2009
Posts: 92883
Own Kudos [?]: 618614 [45]
Given Kudos: 81563
Send PM
Most Helpful Reply
Tutor
Joined: 16 Oct 2010
Posts: 14816
Own Kudos [?]: 64883 [7]
Given Kudos: 426
Location: Pune, India
Send PM
Math Expert
Joined: 02 Sep 2009
Posts: 92883
Own Kudos [?]: 618614 [6]
Given Kudos: 81563
Send PM
General Discussion
User avatar
Manager
Manager
Joined: 04 Jan 2014
Posts: 144
Own Kudos [?]: 131 [2]
Given Kudos: 15
GMAT 1: 660 Q48 V32
GMAT 2: 630 Q48 V28
GMAT 3: 680 Q48 V35
Send PM
Re: If m and n are consecutive positive integers, is m greater than n ? [#permalink]
2
Bookmarks
St1: m-1 and n+1 are consecutive positive integers.

Case 1:
m is even, n is odd and m < n
Let m = 2, n = 3
m-1 = 1, n+1 = 4 -> not consecutive positive integers.

Case 2:
m is even, n is odd and m > n
Let m = 2, n = 1
m-1 = 1, n+1 = 2 -> consecutive positive integers.

Case 3:
m is odd, n is even and m < n
Let m = 3, n = 4
m-1 = 2, n+1 = 5 -> not consecutive positive integers.

Case 4:
m is odd, n is even and m > n
Let m = 3, n = 2
m-1 = 2, n+1 = 3 -> consecutive positive integers.

Not sufficient. Dow to B, C or E.

St2: m is an even integer. Not sufficient. If m = 2 and n = 1 -> m > n but if m = 2 and n = 3 -> m < n.

St 1 + St2: Not sufficient as illustrated by case 1 and case 2.

Answer (E).
avatar
Intern
Intern
Joined: 30 Nov 2013
Posts: 19
Own Kudos [?]: 54 [2]
Given Kudos: 6
Send PM
Re: If m and n are consecutive positive integers, is m greater than n ? [#permalink]
2
Kudos
Donnie84 wrote:
St1: m-1 and n+1 are consecutive positive integers.

Case 1:
m is even, n is odd and m < n
Let m = 2, n = 3
m-1 = 1, n+1 = 4 -> not consecutive positive integers.

Case 2:
m is even, n is odd and m > n
Let m = 2, n = 1
m-1 = 1, n+1 = 2 -> consecutive positive integers.

Case 3:
m is odd, n is even and m < n
Let m = 3, n = 4
m-1 = 2, n+1 = 5 -> not consecutive positive integers.

Case 4:
m is odd, n is even and m > n
Let m = 3, n = 2
m-1 = 2, n+1 = 3 -> consecutive positive integers.

Not sufficient. Dow to B, C or E.

St2: m is an even integer. Not sufficient. If m = 2 and n = 1 -> m > n but if m = 2 and n = 3 -> m < n.

St 1 + St2: Not sufficient as illustrated by case 1 and case 2.

Answer (E).


I think you are making a mistake somewhere !! Well from the different cases you have illustrated it is clear that "A" is sufficient !! Because both cases where m-1,n+1 were consecutive you had m>n therefore statement "1" is clearly sufficient. Hope that helped ;)
Manager
Manager
Joined: 23 May 2013
Posts: 170
Own Kudos [?]: 402 [0]
Given Kudos: 42
Location: United States
Concentration: Technology, Healthcare
GMAT 1: 760 Q49 V45
GPA: 3.5
Send PM
Re: If m and n are consecutive positive integers, is m greater than n ? [#permalink]
The question states that \(m\) &\(n\) are consecutive positive integers. The question can then be boiled down to whether the order is \(m,n\) or \(n,m\).

(1) m-1 and n+1 are consecutive positive integers.

We know that \(m-1\) and \(n+1\)are consecutive positive integers. If the order is \(m,n\) then this statement clearly fails. If the order is \(n,m\) then this statement is true, hence \(A\) is sufficient.

(2) m is an even integer.

If \(m\) is an even integer, \(n\) must be odd. \(n\) can still fall to the left or right of \(m\), and therefore statement (2) is insufficient.

Answer: A
RC & DI Moderator
Joined: 02 Aug 2009
Status:Math and DI Expert
Posts: 11161
Own Kudos [?]: 31870 [1]
Given Kudos: 290
Send PM
Re: If m and n are consecutive positive integers, is m greater than n ? [#permalink]
1
Kudos
Expert Reply
Bahadur wrote:
If m and n are consecutive positive integers, is m greater than n ?

(1) m -1 and n + 1 are consecutive positive integers.
(2) m is an even integer.




Arithmetic Properties of numbers
For two integers x and y to be consecutive, it is
both necessary and sufficient that I x - y I = 1.
(1) Given that m-1 and n + 1 are consecutive
integers, it follows that ~ m - 1)- ( n + 1 ~ = 1,
or lm - n - ~ = 1. Therefore, m - n - 2 = 1 or
m - n - 2 = -1. The former equation implies
that m- n = 3, which contradicts the fact
that m and n are consecutive integers.
Therefore, m - n - 2 = -1, or m = n + 1,
and hence m > n; SUFFICIENT.
(2) If m = 2 and n = 1, then m is greater than n.
However, if m = 2 and n = 3, then m is not
greater than n; NOT sufficient

I know all of the topics of the question. But my confusion is that question describes m and n are consecutive positive integers, so why we need
I x - y I = 1. Why we use absolute for positive integers. Would any one like to explain me please.


Hi,
we take ABSOLUTE MOD because we are not aware whether x is big or y is big..
if x is bigger, x-y=1..
if y is bigger, x-y=-1..
so |x-y|=1..
Manager
Manager
Joined: 27 Dec 2016
Posts: 241
Own Kudos [?]: 238 [0]
Given Kudos: 1103
Send PM
Re: If m and n are consecutive positive integers, is m greater than n ? [#permalink]
I have one small confusion. The question says that "if m and n are consecutive positive integers..." Doesn't this imply automatically that the order is m and then n? How are we unsure that its not m and n rather it could be n and m? Please help me.

Thanks!
Math Expert
Joined: 02 Sep 2009
Posts: 92883
Own Kudos [?]: 618614 [0]
Given Kudos: 81563
Send PM
Re: If m and n are consecutive positive integers, is m greater than n ? [#permalink]
Expert Reply
csaluja wrote:
I have one small confusion. The question says that "if m and n are consecutive positive integers..." Doesn't this imply automatically that the order is m and then n? How are we unsure that its not m and n rather it could be n and m? Please help me.

Thanks!


If it were the way you are saying, we won't have this question. So, x and y are consecutive integers does NOT necessarily mean that y > x.

P.S. Also, note that this is an official question.
IIM School Moderator
Joined: 04 Sep 2016
Posts: 1261
Own Kudos [?]: 1238 [1]
Given Kudos: 1207
Location: India
WE:Engineering (Other)
Send PM
Re: If m and n are consecutive positive integers, is m greater than n ? [#permalink]
1
Kudos
VeritasKarishma chetan2u MentorTutoring nick1816

Quote:
If m and n are consecutive positive integers, is m greater than n ?

(1) m-1 and n+1 are consecutive positive integers.
(2) m is an even integer.


I find the highlighted part and St 1 to be conflicting (usually I take the approach if I could SOLVE q stem by using the statements)
If m and n are two consecutive no, are not m-1 and n-1 ALSO consecutive by default?

To avoid, confusion, can I safely use number picking as below:
Case 1: I take m-1 less than n+1
m-1 = 2
n+1 = 3
hence m=3, n=2 m>n

In next case, I purposely take m-1 greater than n+1
m-1= 4
n+1= 3
Hence m=6, n=2 Still m>n

Clearly St 1 is suff.

St 2 is clearly insuff. Ans: A
Can you share your two cents?
RC & DI Moderator
Joined: 02 Aug 2009
Status:Math and DI Expert
Posts: 11161
Own Kudos [?]: 31870 [0]
Given Kudos: 290
Send PM
Re: If m and n are consecutive positive integers, is m greater than n ? [#permalink]
Expert Reply
adkikani wrote:
VeritasKarishma chetan2u MentorTutoring nick1816

Quote:
If m and n are consecutive positive integers, is m greater than n ?

(1) m-1 and n+1 are consecutive positive integers.
(2) m is an even integer.


I find the highlighted part and St 1 to be conflicting (usually I take the approach if I could SOLVE q stem by using the statements)
If m and n are two consecutive no, are not m-1 and n-1 ALSO consecutive by default?

To avoid, confusion, can I safely use number picking as below:
Case 1: I take m-1 less than n+1
m-1 = 2
n+1 = 3
hence m=3, n=2 m>n

In next case, I purposely take m-1 greater than n+1
m-1= 4
n+1= 3
Hence m=6, n=2 Still m>n

Clearly St 1 is suff.

St 2 is clearly insuff. Ans: A
Can you share your two cents?


Yes, if m and n are consecutive, m-1 and n-1 would surely be consecutive, but this is a peculiar case where m-1 and n+1 are also consecutive.
This now should tell you that n+1>m-1 or n+1=(m-1)+1...n+1=m...m>n
Volunteer Expert
Joined: 16 May 2019
Posts: 3512
Own Kudos [?]: 6856 [1]
Given Kudos: 500
Re: If m and n are consecutive positive integers, is m greater than n ? [#permalink]
1
Kudos
Expert Reply
Hello, adkikani. My in-line responses are below.

adkikani wrote:
VeritasKarishma chetan2u MentorTutoring nick1816

Quote:
If m and n are consecutive positive integers, is m greater than n ?

(1) m-1 and n+1 are consecutive positive integers.
(2) m is an even integer.


I find the highlighted part and St 1 to be conflicting (usually I take the approach if I could SOLVE q stem by using the statements)
If m and n are two consecutive no, are not m-1 and n-1 ALSO consecutive by default?

I do not understand the conflict. Statement (1) presents the information in a distorted way, making it such that the larger value has changed places. That is, it is clear that m is greater than n within the constraints of the question stem and this statement, but n + 1 yields the greater value than m - 1. This is the sort of confusion the GMAT™ likes to create, just enough to make test-takers doubt themselves.
adkikani wrote:
To avoid, confusion, can I safely use number picking as below:
Case 1: I take m-1 less than n+1
m-1 = 2
n+1 = 3
hence m=3, n=2 m>n

In next case, I purposely take m-1 greater than n+1
m-1= 4
n+1= 3
Hence m=6, n=2 Still m>n

This second case does not conform to the given constraints. If m and n are consecutive positive integers, then m - 1 cannot be greater than n + 1. Assume, for the sake of argument, that you had originally picked n to be the greater of the two integers. For instance, let n = 2 and m = 1.

m - 1 = 1 - 1 = 0
n + 1 = 2 + 1 = 3

These values do not conform to the information given in Statement (1), so you would know you had leaned in the wrong direction.
adkikani wrote:
Clearly St 1 is suff.

St 2 is clearly insuff. Ans: A
Can you share your two cents?

Yes, clearly Statement (1) is indeed SUFFICIENT, but be careful about testing, willy-nilly, whatever numbers you want, since that sort of method could lead to incorrect conclusions.

I hope that helps. Thank you for bringing my attention to the question.

- Andrew
Intern
Intern
Joined: 04 Jul 2020
Posts: 7
Own Kudos [?]: 15 [5]
Given Kudos: 91
Location: India
Send PM
Re: If m and n are consecutive positive integers, is m greater than n ? [#permalink]
5
Kudos
Question:- If m and n are consecutive positive integers, is m greater than n?
What I inferred> Question is basically asking, is M=N+1? (as M and N are consecutive, Difference between them is 1)

Statement 1: m-1 and n+1 are consecutive positive integers.

Assume; M=6 & N=5
> M-1 = N = 5
> N+1 = M = 6 ...... (Yes, M>N) .. Thus, Statement sufficient

Statement 2: m is an even integer.
>If m is an even integer, n must be odd. m can still fall to the left or right of n, and therefore statement (2) is insufficient.

Answer: A

Is my approach, Correct?
Bunuel VeritasKarishma chetan2u
Tutor
Joined: 16 Oct 2010
Posts: 14816
Own Kudos [?]: 64883 [1]
Given Kudos: 426
Location: Pune, India
Send PM
Re: If m and n are consecutive positive integers, is m greater than n ? [#permalink]
1
Kudos
Expert Reply
Arpit22 wrote:
Question:- If m and n are consecutive positive integers, is m greater than n?
What I inferred> Question is basically asking, is M=N+1? (as M and N are consecutive, Difference between them is 1)

Statement 1: m-1 and n+1 are consecutive positive integers.

Assume; M=6 & N=5
> M-1 = N = 5
> N+1 = M = 6 ...... (Yes, M>N) .. Thus, Statement sufficient

Statement 2: m is an even integer.
>If m is an even integer, n must be odd. m can still fall to the left or right of n, and therefore statement (2) is insufficient.

Answer: A

Is my approach, Correct?
Bunuel VeritasKarishma chetan2u


Yes, though if it holds for one set of values it doesn't mean it will hold for all. Understand the logic I discussed above to know why it will hold for all.
Intern
Intern
Joined: 24 May 2020
Posts: 3
Own Kudos [?]: 6 [0]
Given Kudos: 17
Send PM
Re: If m and n are consecutive positive integers, is m greater than n ? [#permalink]
let's choose one random positive integer as m: exp: m=5 => n could be: 4 or 6

(1) (m-1) and (n+1) are positive consecutive integers
(m-1)= 5-1=4
#1: n=4 => n+1= 5
#2: n=6 => n+1= 6
Keep #1 (4 and 5 are consecutive positive integers) and eliminate #2 based on the (1) statement, we could conclude m>n : SUFFICIENT

(2) m is even => n must be odd. But cannot determine m is consecutively after n or not : INSUFICIENT

===> A
User avatar
Non-Human User
Joined: 09 Sep 2013
Posts: 32633
Own Kudos [?]: 821 [0]
Given Kudos: 0
Send PM
Re: If m and n are consecutive positive integers, is m greater than n ? [#permalink]
Hello from the GMAT Club BumpBot!

Thanks to another GMAT Club member, I have just discovered this valuable topic, yet it had no discussion for over a year. I am now bumping it up - doing my job. I think you may find it valuable (esp those replies with Kudos).

Want to see all other topics I dig out? Follow me (click follow button on profile). You will receive a summary of all topics I bump in your profile area as well as via email.
GMAT Club Bot
Re: If m and n are consecutive positive integers, is m greater than n ? [#permalink]
Moderator:
Math Expert
92883 posts

Powered by phpBB © phpBB Group | Emoji artwork provided by EmojiOne